Question

3. Let N = R and P be the probability distribution on (R, B(R)) with density 1 -131 XER. Put X(w):=w2, WEN. (a) Describe o(X)
0 0
Add a comment Improve this question Transcribed image text
Answer #1

a)

The \sigma-algebra \hspace{0.1 cm} \sigma(X) is set of subsets of \Omega =\mathbb{R}^{2} such that for w \in \sigma(X)=>w^{c}\in \sigma(X) .

b)

Let Y follow a double exponential distribution with parameter 1(that is Y has the pdf given in the question).

Here X=Y^{2}

Jacobian:J=\pm \frac{1}{2\sqrt{x}}

PDF of X:

f_{X}(x)=\frac{1}{4\sqrt{x}}e^{-\sqrt{x}},x\geq 0, and , 0,otherwise

CDF of X:

F_{X}(x)=0,x <0,and ,\int_{0}^{x}f_{X}(t)dt,x\geq 0\newline =0,x <0,and ,\int_{0}^{x}\frac{1}{4\sqrt{t}}e^{-\sqrt{t}}dt,x\geq 0\newline =0,x <0,and ,\int_{0}^{\sqrt{x}}\frac{1}{2}e^{-z}dt,x\geq 0, substituting \hspace{0.1 cm}z=\sqrt{x}\newline =0,x <0,and ,\frac{1}{2}(1-e^{-\sqrt{x}}),x\geq 0

c)

E(X)=\int_{0}^{\infty}\frac{1}{4\sqrt{t}}te^{-\sqrt{t}}dt=\int_{0}^{\infty}\frac{1}{4}\sqrt{t}e^{-\sqrt{t}}dt=\int_{0}^{\infty}\frac{1}{2}z^{2}e^{-z}dt,\newline subtituting\hspace{0.1 cm}z=\sqrt{t},\newline =\frac{\Gamma(3)}{2}=1

E(e^{tX})=\int_{0}^{\infty}\frac{1}{4\sqrt{x}}e^{tx}e^{-\sqrt{x}}dx=\int_{0}^{\infty}\frac{1}{2}e^{tz^{2}-z}dz,\newline subtituting\hspace{0.1 cm}z=\sqrt{t}\newline =\int_{0}^{\infty}\frac{1}{2}e^{-(\sqrt{t}z-\frac{1}{2\sqrt{t}})^{2}}dz=\int_{0}^{\infty}\frac{1}{2}e^{-t(z-\frac{1}{2t})^{2}}dz\newline =\int_{0}^{\infty}\frac{1}{2}e^{-t(z-\frac{1}{2t})^{2}}dz\newline =\frac{1}{2}e^{-\frac{1}{4t}}\frac{1}{\sqrt{2t}}\int_{0}^{\infty}\frac{1}{\sqrt{2\pi \frac{1}{2t}}}e^{-t(z-\frac{1}{2t})^{2}}dz,\frac{1}{\sqrt{2\pi \frac{1}{2t}}}e^{-t(z-\frac{1}{2t})^{2}}\equiv N(\frac{1}{2t},\frac{1}{2t})\newline =\frac{1}{2}e^{-\frac{1}{4t}}\frac{1}{\sqrt{2t}}(\int_{0}^{\frac{1}{2t}}\frac{1}{\sqrt{2\pi \frac{1}{2t}}}e^{-t(z-\frac{1}{2t})^{2}}dz+\int_{\frac{1}{2t}}^{\infty}\frac{1}{\sqrt{2\pi \frac{1}{2t}}}e^{-t(z-\frac{1}{2t})^{2}}dz)\newline

=\frac{1}{2}e^{-\frac{1}{4t}}\frac{1}{\sqrt{2t}}(\Phi(0)-\Phi(-\frac{1}{\sqrt{2t}})+\frac{1}{2})\newline =\frac{1}{2}e^{-\frac{1}{4t}}\frac{1}{\sqrt{2t}}(1-\Phi(-\frac{1}{\sqrt{2t}}))

Add a comment
Know the answer?
Add Answer to:
3. Let N = R and P be the probability distribution on (R, B(R)) with density...
Your Answer:

Post as a guest

Your Name:

What's your source?

Earn Coins

Coins can be redeemed for fabulous gifts.

Not the answer you're looking for? Ask your own homework help question. Our experts will answer your question WITHIN MINUTES for Free.
Similar Homework Help Questions
ADVERTISEMENT
Free Homework Help App
Download From Google Play
Scan Your Homework
to Get Instant Free Answers
Need Online Homework Help?
Ask a Question
Get Answers For Free
Most questions answered within 3 hours.
ADVERTISEMENT
ADVERTISEMENT
ADVERTISEMENT